\documentclass[a5paper,10pt]{article} \usepackage{myXsim} \usepackage{tasks} % Title Page \title{DM2 \hfill BOUCHOUX Kevin} \tribe{TST} \date{\hfillÀ render pour le Mercredi 24 février} \xsimsetup{ solution/print = false } \begin{document} \maketitle \begin{exercise}[subtitle={Loi binomiale}] Trois personnes s'apprêtent à passer le portique de sécurité. On suppose que pour chaque personne la probabilité que le portique sonne est égale à $0.7$. Soit $X$ la variable aléatoire donnant le nombre de personnes faisant sonner le portique, parmi les 3 personnes de ce groupe. \begin{enumerate} \item Tracer l'arbre représentant le situation. \item Justifier que $X$ suit une loi binomiale dont on précisera les paramètres. \item Quelle est la probabilité qu'une seule personne fasse sonner le portique? \item Calculer puis interpréter les probabilités suivantes \[ P(X = 0) \qquad \qquad P(X \geq 2) \] \item Calculer l'espérance de $X$ et interpréter le résultat. \end{enumerate} \end{exercise} \begin{solution} \begin{enumerate} \item \begin{tikzpicture}[sloped] \node {.} child {node {$0$} child {node {$0$} child {node {$0$} edge from parent node[above] {0.3} } child {node {$1$} edge from parent node[above] {0.7} } edge from parent node[above] {0.3} } child[missing] {} child {node {$1$} child {node {$0$} edge from parent node[above] {0.3} } child {node {$1$} edge from parent node[above] {0.7} } edge from parent node[above] {0.3} } edge from parent node[above] {0.3} } child[missing] {} child[missing] {} child[missing] {} child { node {$1$} child {node {$0$} child {node {$0$} edge from parent node[above] {0.3} } child {node {$1$} edge from parent node[above] {0.7} } edge from parent node[above] {0.3} } child[missing] {} child {node {$1$} child {node {$0$} edge from parent node[above] {0.3} } child {node {$1$} edge from parent node[above] {0.7} } edge from parent node[above] {0.3} } edge from parent node[above] {0.7} } ; \end{tikzpicture} \item Chaque personne a 2 possibilités (1: fait sonner ou 2: ne fait pas sonner) et l'on fait passer 3 personnes ce qui correspond à une répétition identique et aléatoire. On peut donc modéliser la situation par une loi binomiale. \[ X \sim \mathcal{B}(3; 0.76) \] \item Probabilité qu'une seule personne fasse sonner le portique. On voit qu'il y a 3 branches qui correspondent à cette situation dont \[ P(X = 1) = 3 \times 0.7^1 \times 0.3^2 \approx 0.189 \] \item \[ P(X = 0) = 0.3^3 \approx 0.027 \] \[ P(X \geq 2) = P(X = 2) + P(X = 3) = 3 \times 0.7^2 \times 0.3^1 + 0.7^3 \approx 0.784 \] \item Il faut d'abord tracer le tableau résumant la loi de probabilité: \begin{center} \begin{tabular}{|c|*{4}{c|}} \hline Valeur & 0 & 1 & 2 & 3 \\ \hline Probabilité & $0.027$ & $0.189$ & $0.441$ &$0.343$ \\ \hline \end{tabular} \end{center} On peut alors calculer l'espérance \[ E[X] = 0 \times 0.027 + 1 \times 0.189 + 2 \times 0.441 + 3 \times 0.343 = 2.1 \] On peut donc estimer qu'il y aura en moyenne $2.1$ personnes qui feront sonner le portique sur les 3 personnes. \end{enumerate} \end{solution} \begin{exercise}[subtitle={Équation puissance}] Résoudre les équations et inéquations suivantes \begin{multicols}{2} \begin{enumerate} \item $10^x = 4$ \item $7^x = 14$ \item $0.44^x \leq 29$ \item $6 \times 0.27^x = 10$ \end{enumerate} \end{multicols} \end{exercise} \begin{solution} Les solutions ci-dessous ne sont pas justifiée car l'ordinateur ne sait pas faire. Par contre, vous vous devez savoir justifier vos réponses! \begin{enumerate} \item $x = \log(4)$ \item $x = \frac{\log(14)}{\log(7)}$ \item Il faut faire attention quand on divise par un log car ce dernier peut être négatif ce qui est le cas ici. Il faut donc pense à changer le sens de l'inégalité. $x \geq \frac{\log(29)}{\log(0.44)}$ \item Il faut penser à faire la division à par $6$ avant d'utiliser le log car sinon, on ne peut pas utiliser la formule $\log(a^n) = n\times \log(a)$. $x = \frac{\log(1.67)}{\log(0.27)}$ \end{enumerate} \end{solution} \begin{exercise}[subtitle={Étude de fonctions}] Soit $f(x) = 10x^3 - 840x^2 + 18450x - 1$ une fonction définie sur $\R$. \begin{enumerate} \item Calculer $f'(x)$ la dérivée de $f(x)$. \item Calculer $f'(41)$ et $f'(15)$. \item En déduire une forme factorisée de $f'(x)$. \item Étudier le signe de $f'(x)$ et en déduire les variations de $f(x)$. \item Est-ce que la fonction $f(x)$ admet un maximum ou un minimum? Si oui, calculer sa valeur. \end{enumerate} \end{exercise} \begin{solution} \begin{enumerate} \item Dérivée de $f(x)$: $f'(x) = 30x^2 - 1680x + 18450$ \item \begin{align*} f'(41) &= 30 \times 41^{2} - 1680 \times 41 + 18450\\&= 30 \times 1681 - 68880 + 18450\\&= 50430 - 50430\\&= 0 \end{align*} \begin{align*} f'(15) &= 30 \times 15^{2} - 1680 \times 15 + 18450\\&= 30 \times 225 - 25200 + 18450\\&= 6750 - 6750\\&= 0 \end{align*} Donc $x = 41$ et $x=15$ sont des racines de $f'(x) = 30x^2 - 1680x + 18450$. \item On en déduit la forme factorisée suivante \[ f'(x) = 30 (x - 41)(x-15) \] \item Pas de correction disponible \item À causes des branches extérieurs, la fonction $f(x)$ n'a pas de maximum ou de minimum. \end{enumerate} \end{solution} %\printsolutionstype{exercise} \end{document} %%% Local Variables: %%% mode: latex %%% TeX-master: "master" %%% End: